Ausgang eines Strahlteilers mit Zustandseingängen für die Photonenzahl (Fock).

Bei einem unten gezeichneten Strahlteiler, wo A ^ Und B ^ sind eingegebene modale Vernichtungsoperatoren, Transmissivität ist τ [ 0 , 1 ] , und ausgabemodale Vernichtungsoperatoren sind C ^ = τ A ^ + 1 τ B ^ Und D ^ = 1 τ A ^ + τ B ^ , nehme die Eingaben an A ^ Und B ^ befinden sich in Photonenzahl(Fock)-Zuständen | M Und | N , bzw. Welche Zustände haben die Ausgänge C ^ Und D ^ ?

Strahlteiler

Ich verstehe das, wenn einer der Eingänge ein Vakuumzustand ist | 0 , dann sind die Ausgangszustände binomiale Mischungen von Photonenzahlzuständen, wobei der Parameter "Erfolgswahrscheinlichkeit" einer von beiden ist τ oder 1 τ und der Parameter "Anzahl der Versuche" die Photonenzahl ist N des Nicht-Vakuumeingangs (also, wenn | 0 wurde im Modus eingegeben A ^ Und | N im Modus B ^ , dann Modus C ^ ist im Staat k = 0 N ( N k ) ( 1 τ ) k τ N k | k und Modus D ^ ist im Staat k = 0 N ( N k ) τ k ( 1 τ ) N k | k ). Ich frage mich, wie sich dies verallgemeinern lässt, wenn sich beide Eingabemodi in den Nicht-Vakuumzuständen befinden.

Und wenn Sie bei einem 50-50-Strahlteiler an jedem Eingang ein Photon eingeben, kommen zwei aus einem Ausgang und keines aus dem anderen Ausgang.
@PeterShor Warum sagst du das?
Es ist der Hong - Ou - Mandel - Effekt.
Leider ist mein Fall nicht so einfach wie der HOM-Effekt ... Strahlteiler nicht unbedingt 50-50 und eine beliebige Anzahl von Photonen kann an jedem Eingang sein ...
@MBM: Mit der gleichen Idee wie beim HOM-Effekt ist es einfach, für ein Photon an jedem Eingang und einen beliebigen Strahlteiler zu berechnen: 2 aus einem Ausgang mit Wahrscheinlichkeit 2 τ ( 1 τ ) ; eine von jeder Ausgabe mit Wahrscheinlichkeit ( 1 2 τ ) 2 . Es wird komplizierter, wenn Sie größere Eingabephotonenzahlen haben. Siehe dieses Papier . Auch diese . Hier ohne Pay-Wall erhältlich .
Ich denke, Sie sollten irgendwo ein Minuszeichen haben, weil Ihre Matrix ( C ^ , D ^ Die Funktion von A ^ , B ^ ) ist nicht einheitlich.
Befolgen Sie die Ratschläge von @PeterShor und korrigieren Sie das Minuszeichenproblem. Invertieren Sie einfach die Matrix, um sie auszudrücken A ^ , B ^ Die Funktion von C ^ , D ^ . Die Matrix ist real, also funktioniert sie auch für die Transformation von Erstellungsoperatoren. Mit einem Partikel drin A , und ein Partikel in B , Sie erhalten, (bis vielleicht zu einem Amplitudenzeichen) , A + B + = τ ( 1 τ ) ( ( D + ) 2 ( C + ) 2 ) ( 2 τ 1 ) C + D + .
@PeterShor Sieht so aus, als ob Gleichung (48a) in der von Ihnen angegebenen Referenz das ist, was ich brauche - danke! Sieht in der Tat kompliziert aus.
Ich denke nicht, dass Ihre Aussage, wenn Modus a Vakuum ist, Modus c in diesem Zustand ist, nicht richtig ist. Das ist unter der Situation, dass der ursprüngliche Zustand ein Tensorprodukt ist N Einzelphotonenzustand, wo die N Photonen sind nicht miteinander verbunden, anstelle des Fock-Zustands, wo N Photonen sind verwandt und untrennbar miteinander verbunden. Eigentlich versuche ich, die folgende Formel zu verwenden, um die Photonenzahlverteilung zu berechnen. Aber es ist zu banal. Vielleicht können wir darüber reden und Erfahrungen austauschen.
P N = T R ( ρ : ( C + C ) N / N ! e C + C : )

Antworten (1)

Die von Ihnen angegebenen Transformationsgleichungen sind nicht korrekt, da sie die Einheitlichkeit nicht berücksichtigen. Die Bedingung der Einheitlichkeit (oder Energieerhaltung) für die Wirkung des Strahlteilers ergibt die folgenden Transformationen:

C ^ = τ A ^ + 1 τ B ^

D ^ = 1 τ A ^ τ B ^

Das Minuszeichen in der zweiten Gleichung stellt sicher, dass die Einheitlichkeit eingehalten wird.

Aus Gründen, die bald klar werden, lassen Sie uns diese Gleichungen umkehren, um die Eingabemodusoperatoren zu erhalten A ^ Und B ^ in Bezug auf die Ausgabemodusoperatoren C ^ Und D ^ . Wie aufgrund von Reversibilitätsargumenten erwartet, erhalten wir:

A ^ = τ C ^ + 1 τ D ^

B ^ = 1 τ C ^ τ D ^

Es ist nützlich, dieses Problem im Heisenberg-Bild zu betrachten, wo die Wirkung des Strahlteilers vollständig auf den Modenerzeugungs- und -vernichtungsoperatoren liegt, wobei der anfängliche Feldzustand als Vakuum angenommen wird.

Da die betrachteten Eingangszustände die Fock-Zustände sind | M A Und | N B Der vollständige anfängliche Feldzustand kann alternativ geschrieben werden als:

( A ) M ( B ) N | 0 A | 0 B | 0 C | 0 D

Jetzt ersetzen wir die früheren Ausdrücke durch A ^ Und B ^ bezüglich C ^ Und D ^ gegeben durch die Strahlteilertransformationen. Der Feldzustand nach den Modustransformationen ist

( τ C ^ + 1 τ D ^ ) M ( 1 τ C ^ τ D ^ ) N | 0 A | 0 B | 0 C | 0 D

Somit wurden die Ausgangszustände für eine Strahlteilertransformation an Eingangs-Fock-Zuständen erhalten.

Wie Peter Shor zu Recht betonte, ist eine schöne Folge dieser Transformationen der Hong-Ou-Mandel-Effekt. Es besagt, dass beim gleichzeitigen Einfallen von Einzelphotonenzuständen auf die Eingangsports des Strahlteilers beide Photonen aus demselben Ausgangsport austreten.

Dies lässt sich leicht anhand der Gleichung überprüfen, die wir durch Setzen erhalten haben M = N = 1 . Lassen Sie uns auch der Einfachheit halber setzen τ = 0,5 dh der Strahlteiler ist 50 : 50 Verhältnis. Der Status des Ausgabefelds ist

1 2 ( C ^ + D ^ ) 1 2 ( C ^ D ^ ) | 0 A | 0 B | 0 C | 0 D

= 1 2 ( ( C ^ ) 2 ( D ^ ) 2 ) | 0 A | 0 B | 0 C | 0 D

= 1 2 ( | 2 C | 0 D | 0 C | 2 D )

Wir sehen also deutlich, dass entweder beide Photonen aus Port austreten C oder beide tauchen aus dem Hafen auf D . Ein solcher Zustand wird als Zwei-Photonen-NOON-Zustand bezeichnet (der Zustand sieht so aus, wenn N = 2), und dieser Effekt ist von größter Bedeutung in linearen optischen Quantencomputerschemata.